Isosceles Triangle related rates problem

Click For Summary
The problem involves a trough shaped like an isosceles triangle, with specific dimensions and a water inflow rate of 14 ft³/min. The user attempted to find the rate at which the water level rises when the water is 8 inches deep but made errors in their calculations. Key advice from responders emphasized the need to express the base as a function of height before differentiating. The user was encouraged to revisit the volume formula and clarify their approach to the differentiation process. Correctly applying these principles is essential for solving the related rates problem accurately.
hks118
Messages
19
Reaction score
0

Homework Statement


A trough is 9 ft long and its ends have the shape of isosceles triangles that are 5 ft across at the top and have a height of 1 ft. If the trough is being filled with water at a rate of 14 ft3/min, how fast is the water level rising when the water is 8 inches deep?

Variables:
b=5 ft
h=1 ft
l=9 ft

Homework Equations


v=(1/2)bhl
dv/dt=14
dh/dt=?
when h=2/3 ft

The Attempt at a Solution


I tried doing it the straightforward way:
v=(1/2)bhl
dv/dt=(1/2)(5)(2/3)(dh/dt)(9)
14=(1/2)(5)(2/3)(dh/dt)(9)
dh/dt=14/15

This is wrong. Then I tried using similar triangles to get the new base to go along with the height of 8 in. For that I got 10/3 ft. So,
v=(1/2)bhl
dv/dt=(1/2)(10/3)(2/3)(dh/dt)(9)
14=(1/2)(10/3)(2/3)(dh/dt)(9)
dh/dt= 7/5

This is also incorrect. I really don't know why. Any help would be greatly appreciated!
 
Physics news on Phys.org
Welcome to PF!

Hi hks118! Welcome to PF! :smile:

You need to write b as a function of h before you differentiate. :wink:
 


tiny-tim said:
Hi hks118! Welcome to PF! :smile:

You need to write b as a function of h before you differentiate. :wink:

Thanks for the reply!

So b=5h correct? but if I plug in my h value, I get 10/3, which was wrong.:confused:
 
(just got up :zzz: …)

What was your equation for dv/dt?
 
tiny-tim said:
(just got up :zzz: …)

What was your equation for dv/dt?


Well, v=1/2bhl so I figured dv/dt=1/2bh(dh/dt)l. Or do I need db/dt and dl/dt as well?
 
hks118 said:
Well, v=1/2bhl so I figured dv/dt=1/2bh(dh/dt)l.

That wouldn't be right even if b was constant. :confused:

Try again. :smile:
 
Question: A clock's minute hand has length 4 and its hour hand has length 3. What is the distance between the tips at the moment when it is increasing most rapidly?(Putnam Exam Question) Answer: Making assumption that both the hands moves at constant angular velocities, the answer is ## \sqrt{7} .## But don't you think this assumption is somewhat doubtful and wrong?

Similar threads

Replies
2
Views
1K
  • · Replies 24 ·
Replies
24
Views
2K
  • · Replies 4 ·
Replies
4
Views
5K
  • · Replies 8 ·
Replies
8
Views
13K
  • · Replies 3 ·
Replies
3
Views
2K
  • · Replies 4 ·
Replies
4
Views
2K
Replies
2
Views
2K
  • · Replies 3 ·
Replies
3
Views
5K
  • · Replies 10 ·
Replies
10
Views
2K
Replies
4
Views
2K